182
GMAT Critical Reasoning - Everything you need to know This slideshow features screenshots from GMAT Prep Now’s entire Critical Reasoning module (consisting of 38 videos). It covers everything you need to know, and it includes 24 practice questions. www.GMATPrepNow.com

GMAT Critical Reasoning - everything you need to know

Embed Size (px)

Citation preview

Page 1: GMAT Critical Reasoning - everything you need to know

GMAT Critical Reasoning - Everything you need to know

This slideshow features screenshots from GMAT Prep Now’s entire Critical Reasoning module (consisting of 38 videos). It covers everything you need to know, and it includes 24 practice questions.

www.GMATPrepNow.com

Page 2: GMAT Critical Reasoning - everything you need to know

www.GMATPrepNow.com

Note: since these slides are just snippets of a full-length video course, there may be times when you’re unable to glean all the relevant information from a particular screenshot. If, at any time, you’d like to watch the entire video on a certain topic, just click on the link at the top of that page, and you’ll be taken that that particular video.

GMAT Critical Reasoning - Everything you need to know

Page 3: GMAT Critical Reasoning - everything you need to know

If you enjoy this learning format, let us know, and we’ll add similar resources to

our SlideShare page

GMAT Critical Reasoning - Everything you need to know

Page 4: GMAT Critical Reasoning - everything you need to know

Introduction to Critical Reasoning (watch the entire video here)

Page 5: GMAT Critical Reasoning - everything you need to know

This lesson features GMAT questions from the GMAT® mini-test.

GMAT® questions are the property of the Graduate Management Admission Council® and are used here with their permission.

Introduction to Critical Reasoning (watch the entire video here)

Page 6: GMAT Critical Reasoning - everything you need to know

• About 1/3 of Verbal questions (13 to 15 questions)

• Batches of 2 or 3 questions

• Approximately 2 minutes per question

• Test your ability to reason effectively

Introduction to Critical Reasoning (watch the entire video here)

Page 7: GMAT Critical Reasoning - everything you need to know

Introduction to Critical Reasoning

The average normal infant born in the United States weighs between twelve and fourteen pounds at the age of three months. Therefore, if a three-month-old child weighs only ten pounds, its weight gain has been below the United States average.

Which of the following indicates a flaw in the reasoning above?

A) Weight is only one measure of normal infant development.

B) Some three-month-old children weigh as much as seventeen pounds.

C) It is possible for a normal child to weigh ten pounds at birth.

D) The phrase "below average" does not necessarily mean insufficient.

E) Average weight gain is not the same as average weight.

Passage

(watch the entire video here)

Page 8: GMAT Critical Reasoning - everything you need to know

Introduction to Critical Reasoning

The average normal infant born in the United States weighs between twelve and fourteen pounds at the age of three months. Therefore, if a three-month-old child weighs only ten pounds, its weight gain has been below the United States average.

Which of the following indicates a flaw in the reasoning above?

A) Weight is only one measure of normal infant development.

B) Some three-month-old children weigh as much as seventeen pounds.

C) It is possible for a normal child to weigh ten pounds at birth.

D) The phrase "below average" does not necessarily mean insufficient.

E) Average weight gain is not the same as average weight.

Passage

Question stem

(watch the entire video here)

Page 9: GMAT Critical Reasoning - everything you need to know

Introduction to Critical Reasoning

The average normal infant born in the United States weighs between twelve and fourteen pounds at the age of three months. Therefore, if a three-month-old child weighs only ten pounds, its weight gain has been below the United States average.

Which of the following indicates a flaw in the reasoning above?

A) Weight is only one measure of normal infant development.

B) Some three-month-old children weigh as much as seventeen pounds.

C) It is possible for a normal child to weigh ten pounds at birth.

D) The phrase "below average" does not necessarily mean insufficient.

E) Average weight gain is not the same as average weight.

Passage

Question stem

Answer choices

(watch the entire video here)

Page 10: GMAT Critical Reasoning - everything you need to know

Introduction to Critical Reasoning

• About 1/3 of Verbal questions (13 to 15 questions)

• Batches of 2 or 3 questions

• Approximately 2 minutes per question

• Test your ability to reason effectively

• Arguments consist of premises and a conclusion

• Conclusion: what the author is trying to convince you of

• Premises: the evidence used to support the conclusion

(watch the entire video here)

Page 11: GMAT Critical Reasoning - everything you need to know

Introduction to Critical Reasoning

Premise: All men are mortal Premise: Socrates is a man

All men are mortal. Socrates is a man. Therefore, Socrates is mortal.

Conclusion: Socrates is mortal

+

Premise: Gary late for last 3 days Premise: Flat tire this morning

Conclusion: Gary will be late today

+

Strength of an Argument: How well the conclusion follows from the premises

Inductive: the conclusion is not guaranteed

Deductive: the conclusion is guaranteed

For the past 3 days, Gary has arrived late for work. This morning, while Gary was driving to work, his car got a flat tire.

Therefore, Gary will be late for work today.

(watch the entire video here)

Page 12: GMAT Critical Reasoning - everything you need to know

Introduction to Critical Reasoning

Strength of an Argument: How well the conclusion follows from the premises

Strengthen

Strengthen

For the past 3 days, Gary has arrived late for work. This morning, while Gary was driving to work, his car got a flat tire.

Therefore, Gary will be late for work today.

For the past 53 days, Gary has arrived late for work. This morning, while Gary

was driving to work, his car got a flat tire. Therefore, Gary will be late for work today.

For the past 53 days, Gary has arrived late for work. This morning, while Gary was driving to work, his car got 4 flat

tires, and his engine exploded. Therefore, Gary will be late for work today.

(watch the entire video here)

Page 13: GMAT Critical Reasoning - everything you need to know

Introduction to Critical Reasoning

Strength of an Argument: How well the conclusion follows from the premises

Weaken

For the past 3 days, Gary has arrived late for work. This morning, while Gary was driving to work, his car got a flat tire.

Therefore, Gary will be late for work today.

For the past 3 days, Gary has arrived late for work. This morning, while Gary was driving to work, his car got a flat tire. Gary’s house is 1 block from work.

Therefore, Gary will be late for work today.

(watch the entire video here)

Page 14: GMAT Critical Reasoning - everything you need to know

Introduction to Critical Reasoning

Strength of an Argument: How well the conclusion follows from the premises

• The conclusions in most arguments are not guaranteed

• Most arguments can be strengthened or weakened by adding/altering premises

For the past 3 days, Gary has arrived late for work. This morning, while Gary was driving to work, his car got a flat tire.

Therefore, Gary will be late for work today.

(watch the entire video here)

Page 15: GMAT Critical Reasoning - everything you need to know

Introduction to Critical Reasoning

Strength of an Argument

How well the conclusion follows from the premises

Accept all premises as true!

For the past 3 days, Gary has arrived late for work. This morning, while Gary was driving to work, his car got a flat tire.

Therefore, Gary will be late for work today.

(watch the entire video here)

Page 16: GMAT Critical Reasoning - everything you need to know

Introduction to Critical Reasoning

All pigs can fly. Mount Everest is a pig. Therefore, Mount Everest can fly.

Accept all premises as true!

(watch the entire video here)

Page 17: GMAT Critical Reasoning - everything you need to know

Introduction to Critical Reasoning

All pigs can fly. Mount Everest is a pig. Therefore, Mount Everest cannot fly.

Accept all premises as true!

(watch the entire video here)

Page 18: GMAT Critical Reasoning - everything you need to know

Introduction to Critical Reasoning

• About 1/3 of Verbal questions (13 to 15 questions)

• Batches of 2 or 3 questions

• Approximately 2 minutes per question

• Test your ability to reason effectively

• Arguments consist of premises and a conclusion

• Conclusion: what the author is trying to convince you of

• Premises: the evidence used to support the conclusion

• Assumption(s): unstated premise(s) necessary to reach conclusion

(watch the entire video here)

Page 19: GMAT Critical Reasoning - everything you need to know

Introduction to Critical Reasoning

Assumptions: • Gary’s history of lateness will be repeated today

• The time needed to change tire will exceed the time before work starts

• Gary has no other means by which to arrive at work on time

For the past 3 days, Gary has arrived late for work. This morning, while Gary was driving to work, his car got a flat tire.

Therefore, Gary will be late for work today.

(watch the entire video here)

Page 20: GMAT Critical Reasoning - everything you need to know

Introduction to Critical Reasoning

Premise(s)

Conclusion

+ Assumption(s)

(watch the entire video here)

Page 21: GMAT Critical Reasoning - everything you need to know

Introduction to Critical Reasoning

• About 1/3 of Verbal questions (13 to 15 questions)

• Batches of 2 or 3 questions

• Approximately 2 minutes per question

• Test your ability to reason effectively

• Arguments consist of premises and a conclusion

• Conclusion: what the author is trying to convince you of

• Premises: the evidence used to support the conclusion

• Assumption(s): unstated premise(s) necessary to reach conclusion

Strength of an Argument: How well the conclusion follows

from the premises

Accept all premises as true!

(watch the entire video here)

Page 23: GMAT Critical Reasoning - everything you need to know

Dissecting an Argument

Every hockey fan I know is nice. I do not know Judy, but since she is wearing a hockey jersey, she must be nice.

Summarize: - the conclusion

- the premises

- any assumptions

(watch the entire video here)

Page 24: GMAT Critical Reasoning - everything you need to know

Dissecting an Argument

“Is this what the author is trying to convince me of?”

Every hockey fan I know is nice. I do not know Judy, but since she is wearing a hockey jersey, she must be nice.

Conclusion: J is nice

(watch the entire video here)

Page 25: GMAT Critical Reasoning - everything you need to know

Dissecting an Argument

Every hockey fan I know is nice. I do not know Judy, but since she is wearing a hockey jersey, she must be nice.

Goal: make your summaries as brief as possible (while maintaining coherence)

Conclusion: J is nice

(watch the entire video here)

Page 26: GMAT Critical Reasoning - everything you need to know

Dissecting an Argument

Conclusion: J is nice

Every hockey fan I know is nice. I do not know Judy, but since she is wearing a hockey jersey, she must be nice.

Premise: All H-fans I know are nice Premise: I don’t know J Premise: J wearing jersey

+ Assumption: The H-fans I know are representative Assumption: Wearing jersey makes one a fan

(watch the entire video here)

Page 27: GMAT Critical Reasoning - everything you need to know

Dissecting an Argument

Researcher: Two years ago, a wolf pack was relocated to Bilford Island. Although the local rabbit population has decreased drastically since the relocation, the wolves are not to blame for this decrease. Our study shows that the unprecedented number of recent rabbit deaths is due to the myxoma virus.

Summarize: - the conclusion

- the premises

- any assumptions

(watch the entire video here)

Page 28: GMAT Critical Reasoning - everything you need to know

Dissecting an Argument

Premise-Therefore-Conclusion test

Researcher: Two years ago, a wolf pack was relocated to Bilford Island. Although the local rabbit population has decreased drastically since the relocation, the wolves are not to blame for this decrease. Our study shows that the unprecedented number of recent rabbit deaths is due to the myxoma virus.

Option A: wolves not to blame therefore virus killed rabbits

Option B: virus killed rabbits therefore wolves not to blame

(watch the entire video here)

Page 29: GMAT Critical Reasoning - everything you need to know

Dissecting an Argument

Conclusion: rabbit pop. not wolves’ fault

+

Researcher: Two years ago, a wolf pack was relocated to Bilford Island. Although the local rabbit population has decreased drastically since the relocation, the wolves are not to blame for this decrease. Our study shows that the unprecedented number of recent rabbit deaths is due to the myxoma virus.

Premise: wolves arrived 2 yrs ago Premise: rabbit pop. since Premise: virus caused deaths

Assumption: wolves didn’t contribute to virus

(watch the entire video here)

Page 30: GMAT Critical Reasoning - everything you need to know

Dissecting an Argument

Tips for Identifying Conclusions and Premises

• Watch for trigger words that indicate a conclusion

• Watch for trigger words that indicate a premise

therefore, thus, hence, so, implies, indicates, consequently, as a result, clearly, accordingly, infer, conclude

since, because, for, due to, evidence, on the basis of, given that

• Beware of common argument structures

1. Premise, premise, . . . , conclusion

2. Conclusion, premise, premise . . .

3. Conclusion in the question stem

(watch the entire video here)

Page 31: GMAT Critical Reasoning - everything you need to know

Dissecting an Argument

Summarize: - the conclusion

- the premises

- any assumptions

Until recently, the only fish species living in Chilliwack Lake was the Gigafish. Last month, however, several Sovkafish were spotted in the lake. Unlike Gigafish, Sovkafish do not eat insects; instead, they survive by eating other fish. In other lakes where Sovkafish exist, their populations are limited by Dragonfish, which like to feed on Sovkafish.

Which of the following, if true, most effectively challenges the conclusion that releasing 100 Dragonfish into Chilliwack Lake will allow the Gigafish in Chilliwack Lake to survive?

(watch the entire video here)

Page 32: GMAT Critical Reasoning - everything you need to know

Dissecting an Argument

Until recently, the only fish species living in Chilliwack Lake was the Gigafish. Last month, however, several Sovkafish were spotted in the lake. Unlike Gigafish, Sovkafish do not eat insects; instead, they survive by eating other fish. In other lakes where Sovkafish exist, their populations are limited by Dragonfish, which like to feed on Sovkafish.

Which of the following, if true, most effectively challenges the conclusion that releasing 100 Dragonfish into Chilliwack Lake will allow the Gigafish in Chilliwack Lake to survive?

Conclusion: Releasing Dragons will let Gigas live

+

Premise: Giga were only fish in lake Premise: Sovkas now in lake Premise: Sovkas eat other fish Premise: Dragons eat Sovkas

Assumption: Dragons won't eat the Gigas Assumption: Dragons won't somehow jeopardize Gigas Assumption: 100 Dragons is sufficient

(watch the entire video here)

Page 33: GMAT Critical Reasoning - everything you need to know

Dissecting an Argument

Premise-Therefore-Conclusion test

Tips for Identifying Conclusions and Premises

• Watch for trigger words that indicate a conclusion

• Watch for trigger words that indicate a premise

therefore, thus, hence, so, implies, indicates, consequently, as a result, clearly, accordingly, infer, conclude

since, because, for, due to, evidence, on the basis of, given that

• Beware of common argument structures

1. Premise, premise, . . . , conclusion

2. Conclusion, premise, premise . . .

3. Conclusion in the question stem

(watch the entire video here)

Page 35: GMAT Critical Reasoning - everything you need to know

Common Argument Types

3 Most Common Argument Types

• Cause and Effect

• Statistical

• Analogy

(watch the entire video here)

Page 36: GMAT Critical Reasoning - everything you need to know

Common Argument Types

Cause and Effect

Conclusion: X causes Y

+

Premise: Event X occurs Premise: Event Y occurs

Assumption: X is the only possible cause of Y

A recent study reveals that the rate of obesity is higher among senior citizens who watch more than 8 hours of television per day than among senior citizens who watch fewer than 8 hours of television per day. Therefore, obesity among senior citizens is caused by watching more than 8 hours of television per day.

correlation ≠ causation

Weaken

• Something else causes Y

• Y causes X

• X and Y are coincidental

Strengthen

• More information

• Eliminate other causes of Y

(watch the entire video here)

Page 37: GMAT Critical Reasoning - everything you need to know

Common Argument Types

Statistical

Conclusion: Something about entire population

+ Premise: Information from sample

Assumption: Sample represents entire population

In a recent survey, participants at a Republicans-only dance competition were given a questionnaire. Most of the respondents indicated that they enjoyed singing. Therefore, it can be concluded that most Republicans are outgoing people.

Weaken

• Sample not representative

• Conclusion doesn’t match stats

• Flaw in calculations

Strengthen

• Sample is representative

(watch the entire video here)

Page 38: GMAT Critical Reasoning - everything you need to know

Common Argument Types

Analogy

Conclusion: Some other similarity exists

+

Premise: Similarity between X and Y Premise: Similarity between X and Y Premise: Similarity between X and Y

Assumption: Sharing some means sharing all

Country X is a democratic, tropical country with a population of 5 million, and Country Y is a democratic, tropical country with a population of 5 million. Since Country X is experiencing widespread crop failures, Country Y must be experiencing widespread crop failures as well.

Weaken

• Entities less similar

Strengthen

• Entities even more similar

(watch the entire video here)

Page 39: GMAT Critical Reasoning - everything you need to know

Common Argument Types

Cause and Effect

Conclusion: X causes Y

+

Premise: Event X occurs Premise: Event Y occurs

Assumption: X is the only possible cause of Y

Weaken

• Something else causes Y • Y causes X • X and Y are coincidental

Strengthen

• More information • Eliminate other causes of Y

Statistical

Conclusion: Something about entire population

+ Premise: Information from sample

Assumption: Sample represents entire population

Weaken

• Sample not representative • Conclusion doesn’t match stats • Flaw in calculations

Strengthen

• Sample is representative

Analogy

Conclusion: Some other similarity exists

+

Premise: Similarity between X and Y Premise: Similarity between X and Y Premise: Similarity between X and Y

Assumption: Sharing some means sharing all

Weaken

• Entities less similar

Strengthen

• Entities even more similar

(watch the entire video here)

Page 40: GMAT Critical Reasoning - everything you need to know

General Critical Reasoning Strategy (watch the entire video here)

Page 41: GMAT Critical Reasoning - everything you need to know

Question Types

General Critical Reasoning Strategy

1. Weaken the Argument

2. Strengthen the Argument

3. Assumption

4. Conclusion/Inference

5. Method of Reasoning

6. Flawed Argument

7. Paradox

8. Evaluation

(watch the entire video here)

Page 42: GMAT Critical Reasoning - everything you need to know

General Strategy

General Critical Reasoning Strategy

1. Read the question stem to determine the question type

2. Read the passage (argument) and focus on the required information for that question type

3. Check all answer choices

(watch the entire video here)

Page 43: GMAT Critical Reasoning - everything you need to know

Weaken the Argument Questions (watch the entire video here)

Page 44: GMAT Critical Reasoning - everything you need to know

Weaken the Argument Questions

Conclusion

Premise Premise Premise

Assumption Assumption +

(A) New Premise (B) New Premise (C) New Premise (D) New Premise (E) New Premise

Goal: Find the answer choice that, when added to the argument, undermines the conclusion the most.

(watch the entire video here)

Page 45: GMAT Critical Reasoning - everything you need to know

Weaken the Argument Questions

General Strategy

1. Read the question stem to determine the question type

2. Read the passage (argument) and focus on the required information for that question type

3. Check all answer choices

(watch the entire video here)

Page 46: GMAT Critical Reasoning - everything you need to know

Weaken the Argument Questions

Question stem examples

Which of the new premises hurts the argument the most?

• Which of the following, if true, most seriously weakens the argument above?

• Which of the following, if true, would cast the most serious doubt on the researcher’s conclusion?

• Which of the following, if true, most effectively challenges the conclusion that releasing 100 Dragonfish into Chilliwack Lake will allow the Gigafish in Chilliwack Lake to survive?

• Which of the following, if established, most seriously calls into question the argument above?

• Which of the following, if true, most seriously undermines the spokesperson’s position?

(watch the entire video here)

Page 47: GMAT Critical Reasoning - everything you need to know

Weaken the Argument Questions

General Strategy

1. Read the question stem to determine the question type

2. Read the passage (argument) and focus on the required information for that question type

3. Check all answer choices

(watch the entire video here)

Page 48: GMAT Critical Reasoning - everything you need to know

Weaken the Argument Questions

Weaken the Argument Strategy

1. Identify and summarize the conclusion

2. Identify and summarize the premises

3. Identify any assumptions

4. Check each answer choice while repeating conclusion

5. Check all answer choices

Does this weaken the conclusion that...?

(watch the entire video here)

Page 49: GMAT Critical Reasoning - everything you need to know

Weaken the Argument Questions

Tips

- Undermining an unstated assumption

- Adding a new premise that hurts the conclusion

1. Look for common argument types (cause and effect, statistical, analogy)

2. Common ways to weaken an argument

3. “Weaken” does not necessarily mean “destroy”

4. Beware of answer choices that strengthen the argument

5. Do not try to disprove a premise

6. Goal: Weaken the extent to which the conclusion follows from the premises

(watch the entire video here)

Page 50: GMAT Critical Reasoning - everything you need to know

Weaken the Argument Questions

Weaken the Argument Strategy

1. Identify and summarize the conclusion

2. Identify and summarize the premises

3. Identify any assumptions

4. Check each answer choice while repeating conclusion

5. Check all answer choices

Example conclusion: Eating carrots improves one’s eyesight

Does this weaken the conclusion that...?

(watch the entire video here)

Page 51: GMAT Critical Reasoning - everything you need to know

Practice Question

A program instituted in a particular state allows parents to prepay their children's future college tuition at current rates. The program then pays the tuition annually for the child at any of the state's public colleges in which the child enrolls. Parents should participate in the program as a means of decreasing the cost of their children's college education.

Which of the following, if true, is the most appropriate reason for parents not to participate in the program?

A. The parents are unsure about which public college in the state the child will attend.

B. The amount of money accumulated by putting the prepayment funds in an interest-bearing account today will be greater than the total cost of tuition for any of the public colleges when the child enrolls.

C. The annual cost of tuition at the state's public colleges is expected to increase at a faster rate than the annual increase in the cost of living.

D. Some of the state's public colleges are contemplating large increases in tuition next year.

E. The prepayment plan would not cover the cost of room and board at any of the state's public colleges.

This is a question from the GMAT® mini-test. GMAT® questions are the property of the Graduate Management Admission Council® and are used here with their permission.

Page 52: GMAT Critical Reasoning - everything you need to know

Practice Question

A program instituted in a particular state allows parents to prepay their children's future college tuition at current rates. The program then pays the tuition annually for the child at any of the state's public colleges in which the child enrolls. Parents should participate in the program as a means of decreasing the cost of their children's college education.

Which of the following, if true, is the most appropriate reason for parents not to participate in the program?

P: Program: prepay tuition @ current rates

P: Program pays tuition later

C: Program is cheaper

A: Today’s cost less than future cost

A. The parents are unsure about which public college in the state the child will attend.

B. The amount of money accumulated by putting the prepayment funds in an interest-bearing account today will be greater than the total cost of tuition for any of the public colleges when the child enrolls.

C. The annual cost of tuition at the state's public colleges is expected to increase at a faster rate than the annual increase in the cost of living.

D. Some of the state's public colleges are contemplating large increases in tuition next year.

E. The prepayment plan would not cover the cost of room and board at any of the state's public colleges.

(watch the entire video here)

Page 53: GMAT Critical Reasoning - everything you need to know

Practice Question

To attract members away from other fitness clubs in the city, FitnessTown recently launched a new marketing campaign in which each new member receives a free locker rental, free fitness consultations, and unlimited use of the tanning beds. FitnessTown’s marketing team claims that the campaign has been a huge success, since its membership has already increased 20 percent in the first two weeks of the campaign.

Which of the following, if true, would best help to refute the claim of FitnessTown’s marketing team?

A. Almost all of FitnessTown’s new members had never visited a fitness club before they were enticed by FitnessTown’s new campaign.

B. Most of FitnessTown’s new members do not intend to use the free services offered in the campaign.

C. FitnessTown’s investors hope to increase membership by 50 percent within the first month of the campaign.

D. FitnessTown’s new marketing campaign is identical to its previous marketing campaigns.

E. Studies show that people typically cancel their fitness club memberships within the first year.

Page 54: GMAT Critical Reasoning - everything you need to know

Practice Question

To attract members away from other fitness clubs in the city, FitnessTown recently launched a new marketing campaign in which each new member receives a free locker rental, free fitness consultations, and unlimited use of the tanning beds. FitnessTown’s marketing team claims that the campaign has been a huge success, since its membership has already increased 20 percent in the first two weeks of the campaign.

Which of the following, if true, would best help to refute the claim of FitnessTown’s marketing team?

A. Almost all of FitnessTown’s new members had never visited a fitness club before they were enticed by FitnessTown’s new campaign.

B. Most of FitnessTown’s new members do not intend to use the free services offered in the campaign.

C. FitnessTown’s investors hope to increase membership by 50 percent within the first month of the campaign.

D. FitnessTown’s new marketing campaign is identical to its previous marketing campaigns.

E. Studies show that people typically cancel their fitness club memberships within the first year.

P: Campaign = locker, consult. & tanning

P: Membership up 20% in 2 wks

C: FitnessTown stole others’ members

A: membership increase means stolen

customers.

(watch the entire video here)

Page 55: GMAT Critical Reasoning - everything you need to know

Strengthen the Argument Questions (watch the entire video here)

Page 56: GMAT Critical Reasoning - everything you need to know

Conclusion

Premise Premise Premise

Assumption Assumption +

(A) New Premise (B) New Premise (C) New Premise (D) New Premise (E) New Premise

Goal: Find the answer choice that, when added to the argument, strengthens the conclusion the most.

Strengthen the Argument Questions (watch the entire video here)

Page 57: GMAT Critical Reasoning - everything you need to know

Question stem examples

Which of the new premises improves the argument the most?

Strengthen the Argument Questions

• Which of the following, if true, would provide the most support for the conclusion of the argument above?

• Which of the following statements, if true, most strengthens the author’s argument?

• Which of the following, if true, provides the best indication that Suki’s decision was logically sound?

• Which of the following, if true, most strongly supports the recommendation made by the argument?

(watch the entire video here)

Page 58: GMAT Critical Reasoning - everything you need to know

Does this strengthen the conclusion that...?

Example conclusion: Antonio makes the world’s best spaghetti

Strengthen the Argument Strategy

Strengthen the Argument Questions

1. Identify and summarize the conclusion

2. Identify and summarize the premises

3. Identify any assumptions

4. Check each answer choice while repeating conclusion

5. Check all answer choices

(watch the entire video here)

Page 59: GMAT Critical Reasoning - everything you need to know

Tips

• Look for common argument types (cause and effect, statistical, analogy)

• Common ways to strengthen an argument

• The goal is not to create a perfect argument

• Beware of answer choices that weaken the argument

• Watch out for answer choices that support a premise but not the conclusion

Strengthen the Argument Questions

- Stating a previously-unstated assumption

- Supporting or elaborating on an existing premise

- Adding a new supporting premise

Bonnie must be great chess player

Avram is world-class sprinter Avram is great chess player Bonnie is world-class sprinter +

(A) Bonnie holds 100m world record (B) . (C) . (D) . (E) .

(watch the entire video here)

Page 60: GMAT Critical Reasoning - everything you need to know

Strengthen the Argument Questions

Does this strengthen the conclusion that...?

Strengthen the Argument Strategy

1. Identify and summarize the conclusion

2. Identify and summarize the premises

3. Identify any assumptions

4. Check each answer choice while repeating conclusion

5. Check all answer choices

(watch the entire video here)

Page 61: GMAT Critical Reasoning - everything you need to know

Practice Question

In the 12 years since the pretzel desalinization plant was built on the Polahoochi River, the salinity of the river has increased from 180 µS/cm to 911 µS/cm in the 10-mile stretch of river downstream from the plant. During the same 12-year period, researchers have observed a sharp decline in the average number of eggs laid by individual polafish, a species native to the Polahoochi River. Given this information, Dr. Kim hypothesizes that high salinity levels adversely affect the reproductive organs of the polafish.

Which of the following, if true, would most strengthen Dr. Kim’s hypothesis?

A. In the same state, many populations of polafish that are not downstream of pretzel desalinization plants have experienced a reduction in egg production.

B. Prior to the construction of the pretzel desalinization plant, the salinity of the 10-mile stretch of river downstream never exceeded 180 µS/cm.

C. Other species of fish in the same 10-mile stretch have experienced a sharp reduction in egg production.

D. In the past 12 years, the salinity of the tributaries flowing into the 10-mile stretch of river downstream from the plant has remained below 180 µS/cm, and the polafish living in these tributaries have not experienced any decline in egg production.

E. In other states, fish downstream from pretzel desalinization plants have experienced declines in egg production.

Page 62: GMAT Critical Reasoning - everything you need to know

Practice Question

In the 12 years since the pretzel desalinization plant was built on the Polahoochi River, the salinity of the river has increased from 180 µS/cm to 911 µS/cm in the 10-mile stretch of river downstream from the plant. During the same 12-year period, researchers have observed a sharp decline in the average number of eggs laid by individual polafish, a species native to the Polahoochi River. Given this information, Dr. Kim hypothesizes that high salinity levels adversely affect the reproductive organs of the polafish.

Which of the following, if true, would most strengthen Dr. Kim’s hypothesis?

P: plant blt 12 yrs ago

P: downstream: salt

P: downstream: polafish eggs

C: salinity lowers polafish egg #s

A: salinity is only possible cause of

egg

A. In the same state, many populations of polafish that are not downstream of pretzel desalinization plants have experienced a reduction in egg production.

B. Prior to the construction of the pretzel desalinization plant, the salinity of the 10-mile stretch of river downstream never exceeded 180 µS/cm.

C. Other species of fish in the same 10-mile stretch have experienced a sharp reduction in egg production.

D. In the past 12 years, the salinity of the tributaries flowing into the 10-mile stretch of river downstream from the plant has remained below 180 µS/cm, and the polafish living in these tributaries have not experienced any decline in egg production.

E. In other states, fish downstream from pretzel desalinization plants have experienced declines in egg production.

(watch the entire video here)

Page 64: GMAT Critical Reasoning - everything you need to know

Conclusion

Premise Premise Premise

Assumption Assumption +

(A) Assumption (B) Assumption (C) Assumption (D) Assumption (E) Assumption

Goal: Find a necessary assumption

Assumption Questions (watch the entire video here)

Page 65: GMAT Critical Reasoning - everything you need to know

Question stem examples

Assumption Questions

• Which of the following is an assumption on which the argument depends?

• The scientist’s argument depends on the assumption that

• The conclusion above follows logically if which of the following is assumed?

(watch the entire video here)

Page 66: GMAT Critical Reasoning - everything you need to know

Is this assumption necessary to draw the conclusion that...?

Assumption Question Strategy

1. Identify and summarize the conclusion

2. Identify and summarize the premises

3. Identify any assumptions

4. Look for one of your assumptions among the answer choices

5. Check each answer choice against the conclusion

Assumption Questions

Example conclusion: Hotdogs are bad for one’s health

(watch the entire video here)

Page 67: GMAT Critical Reasoning - everything you need to know

Strengthen the Argument Questions

Assumption Questions

Assumption Questions

Want information that helps the conclusion

Want information that is necessary for the conclusion

(watch the entire video here)

Page 68: GMAT Critical Reasoning - everything you need to know

Negation Technique

Assumption Questions

Basis: 1) An assumption is absolutely necessary for a conclusion to follow from the premises

2) Negating a necessary assumption will destroy the argument

Juan has been practicing tennis 3 hours each day for the past 2 years. Therefore, Juan will win the city championship next month.

C: J will win championship

P: J practicing 3hr/day for 2 yrs

+ A: Nothing stops championship A: J is eligible to play

A: It is not the case that J lives until championship

A: J lives until championship

P: J dies before championship

P: J dies before championship

(watch the entire video here)

Page 69: GMAT Critical Reasoning - everything you need to know

Is this assumption necessary to draw the conclusion that...?

Assumption Question Strategy

1. Identify and summarize the conclusion

2. Identify and summarize the premises

3. Identify any assumptions

4. Look for one of your assumptions among the answer choices

5. Check each answer choice against the conclusion

6. Apply Negation Technique:

7. Check all answer choices

Assumption Questions

Negate each answer choice and insert it into the argument. The negated answer choice that destroys the argument is the correct answer

(watch the entire video here)

Page 70: GMAT Critical Reasoning - everything you need to know

Tips

• Look for common argument types (cause and effect, statistical, analogy)

• Look for shifts in language between premises and conclusion

• Remember that arguments can have any number of assumptions

Assumption Questions

Juan has been practicing tennis 3 hours each day for the past 2 years. Therefore, Juan will win the city championship next month.

C: J will win championship

P: J practicing 3hr/day for 2 mths

+

A: J lives until championship A: Nothing stops championship A: J is eligible to play

(watch the entire video here)

Page 71: GMAT Critical Reasoning - everything you need to know

Practice Question

A. Investment firms should not restrict the ways in which portfolio managers manage their clients’ funds.

B. Portfolio managers can provide above-average returns for their corporate clients only by short selling stocks.

C. Short selling is a technique used primarily for corporate clients.

D. Portfolio managers often used short selling techniques to provide above-average returns for corporate clients.

E. Before the investment firm issued the new rules, portfolio managers were not permitted to short sell stocks.

A leading investment firm has issued new rules that prevent its portfolio managers from short selling stocks for their corporate clients. One portfolio manager has concluded that this restriction prevents portfolio managers from providing above-average investment returns for their corporate clients.

Which of the following is an assumption that would allow the portfolio manager’s conclusion to be properly drawn?

Page 72: GMAT Critical Reasoning - everything you need to know

Practice Question

A. Investment firms should not restrict the ways in which portfolio managers manage their clients’ funds.

B. Portfolio managers can provide above-average returns for their corporate clients only by short selling stocks.

C. Short selling is a technique used primarily for corporate clients.

D. Portfolio managers often used short selling techniques to provide above-average returns for corporate clients.

E. Before the investment firm issued the new rules, portfolio managers were not permitted to short sell stocks.

A leading investment firm has issued new rules that prevent its portfolio managers from short selling stocks for their corporate clients. One portfolio manager has concluded that this restriction prevents portfolio managers from providing above-average investment returns for their corporate clients.

Which of the following is an assumption that would allow the portfolio manager’s conclusion to be properly drawn?

A: SS necessary to make big $

P: New rules prevent SS

C: Rules stop big $ for CC

(watch the entire video here)

Page 73: GMAT Critical Reasoning - everything you need to know

Conclusion/Inference Questions (watch the entire video here)

Page 74: GMAT Critical Reasoning - everything you need to know

?

Premise Premise Premise +

(A) Conclusion (B) Conclusion (C) Conclusion (D) Conclusion (E) Conclusion

Goal: Find conclusion that logically follows

Conclusion/Inference Questions (watch the entire video here)

Page 75: GMAT Critical Reasoning - everything you need to know

Question stem examples

Conclusion/Inference Questions

• The statements above, if true, most strongly support which of the following conclusions?

• If the statements above are true, which of the following must also be true on the basis of them?

• Which of the following hypotheses receives the strongest support from the given information?

• Which of the following can be logically inferred based on the statements above?

Identify something that must follow from the premises

Inference question = Conclusion question

(watch the entire video here)

Page 76: GMAT Critical Reasoning - everything you need to know

Conclusion/Inference Questions

For the past 3 days, all of Florida’s orange farms have experienced freezing temperatures. Therefore, the number of oranges harvested this year will be less than expected.

Typical Conclusion (in most GMAT questions)

Conclusion in a Conclusion question

• Conclusion is partially supported

For the past 3 days, the temperature at every Florida orange farm has not exceeded -5 degrees Celsius.

The statement above, if true, most strongly supports which of the following conclusions?

• Conclusion is guaranteed

(A) The number of oranges harvested this year will be less than expected.

(watch the entire video here)

Page 77: GMAT Critical Reasoning - everything you need to know

Conclusion/Inference Questions

For the past 3 days, all of Florida’s orange farms have experienced freezing temperatures. Therefore, the number of oranges harvested this year will be less than expected.

Typical Conclusion (in most GMAT questions)

Conclusion in a Conclusion question

• Conclusion is partially supported

For the past 3 days, the temperature at every Florida orange farm has not exceeded -5 degrees Celsius.

The statement above, if true, most strongly supports which of the following conclusions?

• Conclusion is guaranteed

(A) The orange harvest at some Florida farms will be less than expected.

(watch the entire video here)

Page 78: GMAT Critical Reasoning - everything you need to know

Conclusion/Inference Questions

For the past 3 days, all of Florida’s orange farms have experienced freezing temperatures. Therefore, the number of oranges harvested this year will be less than expected.

Typical Conclusion (in most GMAT questions)

Conclusion in a Conclusion question

• Conclusion is partially supported

For the past 3 days, the temperature at every Florida orange farm has not exceeded -5 degrees Celsius.

The statement above, if true, most strongly supports which of the following conclusions?

• Conclusion is guaranteed

(A) At least one orange will be damaged from the freezing temperatures.

(watch the entire video here)

Page 79: GMAT Critical Reasoning - everything you need to know

Conclusion/Inference Questions

For the past 3 days, all of Florida’s orange farms have experienced freezing temperatures. Therefore, the number of oranges harvested this year will be less than expected.

Typical Conclusion (in most GMAT questions)

Conclusion in a Conclusion question

• Conclusion is partially supported

For the past 3 days, the temperature at every Florida orange farm has not exceeded -5 degrees Celsius.

The statement above, if true, most strongly supports which of the following conclusions?

• Conclusion is guaranteed

(A) For the past 3 days, not one Florida orange farm has experienced temperatures above -5 degrees Celsius.

(watch the entire video here)

Page 80: GMAT Critical Reasoning - everything you need to know

Conclusion/Inference Questions

For the past 3 days, all of Florida’s orange farms have experienced freezing temperatures. Therefore, the number of oranges harvested this year will be less than expected.

Typical Conclusion (in most GMAT questions)

Conclusion in a Conclusion question

• Conclusion is partially supported

For the past 3 days, the temperature at every Florida orange farm has not exceeded -5 degrees Celsius.

The statement above, if true, most strongly supports which of the following conclusions?

• Conclusion is guaranteed

(A) During the past 3 days, at least one Florida orange farm has experienced temperatures lower than 40 degrees Celsius.

(watch the entire video here)

Page 81: GMAT Critical Reasoning - everything you need to know

Must it be true that...?

Conclusion Question Strategy

1. Identify and summarize the premises

2. Draw a conclusion that must follow

3. Look for your conclusion among the answer choices

4. Aggressively eliminate incorrect answers

5. Apply a version of the Negation Technique:

6. Check all answer choices

Conclusion/Inference Questions

The negated conclusion that contradicts the premises the most is probably the correct answer.

(watch the entire video here)

Page 82: GMAT Critical Reasoning - everything you need to know

Tips

1. Do not stray too far from the premises

2. Look for a rewording of a premise

P: Kyle enjoys chocolate

P: Rome is the capital of Italy

Conclusion/Inference Questions

Example

C: ?

What conclusion can be drawn using every premise?

(watch the entire video here)

Page 83: GMAT Critical Reasoning - everything you need to know

Tips

1. Do not stray too far from the premises

2. Look for a rewording of a premise

P: Kyle enjoys chocolate

P: Rome is the capital of Italy

Conclusion/Inference Questions

Example

C: The capital of Italy is Rome

(watch the entire video here)

Page 84: GMAT Critical Reasoning - everything you need to know

Tips

1. Do not stray too far from the premises

2. Look for a rewording of a premise

3. Conclusions need not involve every premise

4. Do not inject assumptions into the argument

5. Beware of answer choices that introduce new ideas/words

6. Beware of answer choices where the strength of the language does not match the strength of the language in the premises

Conclusion/Inference Questions (watch the entire video here)

Page 85: GMAT Critical Reasoning - everything you need to know

Practice Question

While studying Emperor Penguins in Antarctica during the coldest 3 months of the year, researchers observed that several of the penguins died. The researchers also discovered that the death rate among the larger penguins was greater than the death rate among the smaller penguins. However, the researchers’ conclusion that size was a determinant in these deaths is probably mistaken, since smaller penguins are typically younger than larger ones.

If the above statements are true, which of the following can be inferred?

A. Among Emperor Penguins of the same age, the smaller penguins have a greater survival rate over a 3-month period than the larger penguins do.

B. Among the Emperor Penguins that survived the 3-month period, there is probably no relationship between age and size.

C. Among Emperor Penguins of the same age, a difference in size may not indicate a difference in chances of survival over a 3-month period.

D. Temperature does not play a role in the death rate among Emperor Penguins over a 3-month period.

E. Among Emperor Penguins of the same size, age is a determinant in the survival rate over a 3-month period.

Page 86: GMAT Critical Reasoning - everything you need to know

Practice Question

While studying Emperor Penguins in Antarctica during the coldest 3 months of the year, researchers observed that several of the penguins died. The researchers also discovered that the death rate among the larger penguins was greater than the death rate among the smaller penguins. However, the researchers’ conclusion that size was a determinant in these deaths is probably mistaken, since smaller penguins are typically younger than larger ones.

If the above statements are true, which of the following can be inferred?

P: P’s died during 3 coldest mnths

P: Larger P’s more likely to die

P: Resrchrs conclude size a factor

P: Size probably not a factor

P: Smaller P’s typically younger

A. Among Emperor Penguins of the same age, the smaller penguins have a greater survival rate over a 3-month period than the larger penguins do.

B. Among the Emperor Penguins that survived the 3-month period, there is probably no relationship between age and size.

C. Among Emperor Penguins of the same age, a difference in size may not indicate a difference in chances of survival over a 3-month period.

D. Temperature does not play a role in the death rate among Emperor Penguins over a 3-month period.

E. Among Emperor Penguins of the same size, age is a determinant in the survival rate over a 3-month period.

(watch the entire video here)

Page 88: GMAT Critical Reasoning - everything you need to know

Structure Questions

• Test your understanding of the argumentative strategies employed in an argument

• 3 types of Structure Questions:

- Method of Reasoning

- Boldface

- Parallel Argument

(watch the entire video here)

Page 89: GMAT Critical Reasoning - everything you need to know

(A) Description of argument (B) Description of argument (C) Description of argument (D) Description of argument (E) Description of argument

Goal: Find the best description of the author’s argumentative strategy.

Method of Reasoning questions

Conclusion

Premise Premise Premise

Assumption Assumption +

Structure Questions (watch the entire video here)

Page 90: GMAT Critical Reasoning - everything you need to know

Question stem examples for Method of Reasoning questions

Explain how the author presents his/her argument

Structure Questions

• The author’s point is made by which method of reasoning?

• Which of the following strategies does Dr. Kwan use to defend his position?

• In the passage, the author develops the argument by ___

• The reporter challenges the spokesperson’s position by doing which of the following?

(watch the entire video here)

Page 91: GMAT Critical Reasoning - everything you need to know

Examples of answer choices for Method of Reasoning questions

Structure Questions

• The argument arrives at its conclusion by demonstrating the inherent problems with alternative conclusions.

• The author offers a new definition of a term that is central to an opposing argument.

• The argument employs circular reasoning by assuming that which it is trying to prove.

The answer choices are typically generic

(watch the entire video here)

Page 92: GMAT Critical Reasoning - everything you need to know

Strategy for tackling Method of Reasoning questions

1. Read the passage

2. For each sentence, ask, “What role does this play in the argument?”

3. Identify and summarize the conclusion and premises

4. Use generic language to describe the method of reasoning to yourself

5. Look for your description among the answer choices

6. Check all answer choices

Structure Questions (watch the entire video here)

Page 93: GMAT Critical Reasoning - everything you need to know

(A) Role played by boldfaced portion(s) (B) Role played by boldfaced portion(s) (C) Role played by boldfaced portion(s) (D) Role played by boldfaced portion(s) (E) Role played by boldfaced portion(s)

Goal: Find the best description of the role(s) played

Boldface questions

Conclusion

Premise Premise Premise

Assumption Assumption +

Structure Questions (watch the entire video here)

Page 94: GMAT Critical Reasoning - everything you need to know

Question stem examples for Boldface questions

The passage contains bolded text

Structure Questions

• In the above argument, the portion in boldface plays which of the following roles?

• In the researcher’s argument, the two portions in boldface play which of the following roles?

Researcher: Two years ago, a wolf pack was relocated to Bilford Island. Although the local rabbit population has decreased drastically since the relocation, the wolves are not to blame for this decrease. Our study shows that the unprecedented number of recent rabbit deaths is due to the myxoma virus.

In the above argument, the portion in boldface plays which of the following roles?

(watch the entire video here)

Page 95: GMAT Critical Reasoning - everything you need to know

Strategy for tackling Boldface questions

1. Read the passage

2. For each boldfaced portion, ask, “What role does this play in the argument?”

3. Identify and summarize the conclusion and premises

4. Use generic language to describe the roles played by the boldfaced portion(s)

5. Find the answer choice that most closely matches yours

6. Check all answer choices

Structure Questions (watch the entire video here)

Page 96: GMAT Critical Reasoning - everything you need to know

Tips for Boldface questions

1. Look for common roles:

2. Consider how the second bolded part is related to first bolded part

3. Beware of answer choices that are half right and half wrong

Structure Questions

- Concluding

- Summarizing

- Contradicting

- Providing supporting evidence

- Providing an example

- Providing a counterexample

- Generalizing

(watch the entire video here)

Page 97: GMAT Critical Reasoning - everything you need to know

Structure Questions

(A) Complete argument (B) Complete argument (C) Complete argument (D) Complete argument (E) Complete argument

Goal: Find the argument that employs the most similar argumentative strategy.

Parallel Argument questions

Conclusion

Premise Premise Premise

Assumption Assumption +

(watch the entire video here)

Page 98: GMAT Critical Reasoning - everything you need to know

Structure Questions

Question stem examples for Parallel Argument questions

Find the argument most like the original

• Which of the following arguments exhibits a pattern of reasoning most similar to the pattern of reasoning exhibited in the argument above?

• Which of the following is most like the argument above in its logical structure?

• The pattern of reasoning displayed above is most closely paralleled in which of the following?

(watch the entire video here)

Page 99: GMAT Critical Reasoning - everything you need to know

Structure Questions

Strategy for tackling Parallel Argument questions

1. Identify and summarize the conclusion and premises

2. Use generic language to describe the method of reasoning to yourself before checking the answer choices

3. Look for an argument with same structure

4. Check all answer choices

(watch the entire video here)

Page 100: GMAT Critical Reasoning - everything you need to know

Structure Questions

Tips for Parallel Argument questions

1. Beware of answer choices with same subject matter

2. Questions are time-consuming check your time

(watch the entire video here)

Page 101: GMAT Critical Reasoning - everything you need to know

Structure Questions

• Test your understanding of the argumentative strategies employed in an argument

• 3 types of Structure Questions:

- Method of Reasoning

- Boldfaced

- Parallel Argument

(watch the entire video here)

Page 102: GMAT Critical Reasoning - everything you need to know

Practice Question

ABC Widget’s plan to decrease employee salaries by 15% makes sense. Granted, the reduced salaries will place a financial burden on many employees. But, if the company does not decrease employee salaries, it will not have enough money to stay in business, and everyone at the company will lose his or her job.

The passage employs which of the following argumentative strategies?

A. It explains why the alternative course of action would not be subject to the objections raised against the proposed course of action.

B. It describes an ideal situation by way of a situation that is less than ideal.

C. It indirectly arrives at its conclusion by providing grounds to reject an alternative approach.

D. It extrapolates the perceived outcome of the proposed plan in order to criticize the alternative course of action.

E. It employs circular reasoning by assuming that which it is trying to prove.

Page 103: GMAT Critical Reasoning - everything you need to know

Practice Question

A. It explains why the alternative course of action would not be subject to the objections raised against the proposed course of action.

B. It describes an ideal situation by way of a situation that is less than ideal.

C. It indirectly arrives at its conclusion by providing grounds to reject an alternative approach.

D. It extrapolates the perceived outcome of the proposed plan in order to criticize the alternative course of action.

E. It employs circular reasoning by assuming that which it is trying to prove.

P: 15% cut will hurt many

P: No cut no jobs

C: 15% cut makes sense

Not doing X is much worse than doing X. So, we should do X.

ABC Widget’s plan to decrease employee salaries by 15% makes sense. Granted, the reduced salaries will place a financial burden on many employees. But, if the company does not decrease employee salaries, it will not have enough money to stay in business, and everyone at the company will lose his or her job.

The passage employs which of the following argumentative strategies?

(watch the entire video here)

Page 104: GMAT Critical Reasoning - everything you need to know

Practice Question

Fred: Columbus was a great explorer because he held on to his conviction that the Earth was round in the face of overwhelming opposition.

Stan: The mark of a great explorer is bravery not the adherence to a conviction. Besides, the concept of a round Earth was widely accepted in the fifteenth century, when Columbus was looking for someone to fund his voyage.

In Stan’s response, the two boldface portions play which of the following

roles?

A. The first presents an alternative conclusion; the second provides evidence in support of that conclusion.

B. The first rejects the criterion on which Fred’s argument is based; the second disputes a specific claim.

C. The first presents an alternative criterion; the second describes a premise on which Stan’s conclusion relies.

D. The first elaborates on Fred’s criterion; the second presents a premise on which Stan’s conclusion relies.

E. The first reveals a contradiction in Fred’s argument; the second resolves that contradiction.

Page 105: GMAT Critical Reasoning - everything you need to know

Practice Question

Fred: Columbus was a great explorer because he held on to his conviction that the Earth was round in the face of overwhelming opposition.

Stan: The mark of a great explorer is bravery not the adherence to a conviction. Besides, the concept of a round Earth was widely accepted in the fifteenth century, when Columbus was looking for someone to fund his voyage.

In Stan’s response, the two boldface portions play which of the following

roles?

A. The first presents an alternative conclusion; the second provides evidence in support of that conclusion.

B. The first rejects the criterion on which Fred’s argument is based; the second disputes a specific claim.

C. The first presents an alternative criterion; the second describes a premise on which Stan’s conclusion relies.

D. The first elaborates on Fred’s criterion; the second presents a premise on which Stan’s conclusion relies.

E. The first reveals a contradiction in Fred’s argument; the second resolves that contradiction.

Fred

P: Conviction despite opposition

C: Columbus great explorer

Stan

1st: Great = bravery not conviction

2nd: Round earth known

Stan

1st: Refutes standard

2nd: Calls statement false

(watch the entire video here)

Page 106: GMAT Critical Reasoning - everything you need to know

Practice Question

In Townville, most smokers play tennis, and most nonsmokers do not play tennis. Therefore, in Townville, most tennis players smoke.

Which of the following exhibits a pattern of flawed reasoning most similar to that in the argument above?

A. In Townville, most Lions Club members were born in Townville, and most of the residents who are not Lions Club members were not born in Townville. Therefore, most of the residents who were born in Townville are Lions Club members.

B. In Townville, most of the people who live west of Main Street own a GPS, and most of the people who own a GPS live east of Main street. Therefore, most of the people in Townville own a GPS.

C. In Townville, most cat owners own exactly one dog, and most dog owners own more than one dog. Therefore, most of the people in Townville who own more than one dog do not own any cats.

D. In Townville, most tennis players play golf, but not every golfer plays tennis. Therefore, in Townville, there are more tennis players than golfers.

E. In Townville, most of the houses are painted red, and most of the houses have a pool. Therefore, in Townville, most of the houses are painted red and have a pool.

Page 107: GMAT Critical Reasoning - everything you need to know

Practice Question

In Townville, most smokers play tennis, and most nonsmokers do not play tennis. Therefore, in Townville, most tennis players smoke.

Which of the following exhibits a pattern of flawed reasoning most similar to that in the argument above?

P: S T

P: ~S ~T

C: T S

A. In Townville, most Lions Club members were born in Townville, and most of the residents who are not Lions Club members were not born in Townville. Therefore, most of the residents who were born in Townville are Lions Club members.

B. In Townville, most of the people who live west of Main Street own a GPS, and most of the people who own a GPS live east of Main street. Therefore, most of the people in Townville own a GPS.

C. In Townville, most cat owners own exactly one dog, and most dog owners own more than one dog. Therefore, most of the people in Townville who own more than one dog do not own any cats.

D. In Townville, most tennis players play golf, but not every golfer plays tennis. Therefore, in Townville, there are more tennis players than golfers.

E. In Townville, most of the houses are painted red, and most of the houses have a pool. Therefore, in Townville, most of the houses are painted red and have a pool.

(watch the entire video here)

Page 108: GMAT Critical Reasoning - everything you need to know

Flawed Argument Questions (watch the entire video here)

Page 109: GMAT Critical Reasoning - everything you need to know

(A) Main problem (B) Main problem (C) Main problem (D) Main problem (E) Main problem

Goal: Find the argument’s primary flaw

Flawed Conclusion

Premise Premise Premise

Assumption Assumption +

Flawed Argument Questions (watch the entire video here)

Page 110: GMAT Critical Reasoning - everything you need to know

Question stem examples

Identify the main problem with the argument

Flawed Argument Questions

• Which of the following identifies the most serious logical flaw in the argument above?

• Which one of the following best identifies the error in reasoning made in the passage?

• The argument is vulnerable to criticism on which one of the following grounds?

• The reasoning in the argument is not sound because it fails to establish that ___

(watch the entire video here)

Page 111: GMAT Critical Reasoning - everything you need to know

1. Identify and summarize the conclusion and premises

2. Identify any unstated assumptions

3. Determine the primary flaw

4. Look for your answer among the answer choices

5. Check all answer choices

Flawed Argument Questions

Strategy

(watch the entire video here)

Page 112: GMAT Critical Reasoning - everything you need to know

Common Flaws

Flawed Argument Questions

While studying the reading levels of elementary students, researchers discovered that, on average, the longer a child’s foot, the higher his/her reading level. Therefore, foot growth increases one’s reading level.

• Confusing causation with correlation

(watch the entire video here)

Page 113: GMAT Critical Reasoning - everything you need to know

Common Flaws

• Confusing causation with correlation

• Confusing numbers with rates

Flawed Argument Questions

Last year, 10 people were murdered in Happyton, and 100 people were murdered in Killington. Therefore, it is much safer to live in Happyton than in Killington

(watch the entire video here)

Page 114: GMAT Critical Reasoning - everything you need to know

Common Flaws

• Confusing causation with correlation

• Confusing numbers with rates

• Conclusion mismatch

Flawed Argument Questions

Last year, hundreds of postal workers in Maltania suffered back injuries from carrying heavy packages at the sorting stations. To help reduce these injuries, the government should introduce a law that prohibits people from mailing packages that are over 3 feet in length.

- Watch out for new words in the conclusion

(watch the entire video here)

Page 115: GMAT Critical Reasoning - everything you need to know

Common Flaws

• Confusing causation with correlation

• Confusing numbers with rates

• Conclusion mismatch

Flawed Argument Questions

• Extreme conclusion

- Watch out for new words in the conclusion

In some countries, raising the minimum wage can help boost the economy. So, if Maltania raises its minimum wage, its economy will improve.

(watch the entire video here)

Page 116: GMAT Critical Reasoning - everything you need to know

Common Flaws

• Confusing causation with correlation

• Confusing numbers with rates

• Conclusion mismatch

Flawed Argument Questions

• Extreme conclusion

• Mistaking necessary for sufficient

- Watch out for new words in the conclusion

In order to be a great science teacher, one must have a deep understanding of biology. Since Terrence has a deep understanding of biology, he must be a great science teacher.

(watch the entire video here)

Page 117: GMAT Critical Reasoning - everything you need to know

Common Flaws

• Confusing causation with correlation

• Confusing numbers with rates

• Conclusion mismatch

Flawed Argument Questions

• Extreme conclusion

• Mistaking necessary for sufficient

• Guilty by association

- Watch out for new words in the conclusion

Sharon, Margaret and Rena are tall, Hoopton High students who play basketball. Since Maureen is a tall, Hoopton High student, she must play basketball.

(watch the entire video here)

Page 118: GMAT Critical Reasoning - everything you need to know

Common Flaws

• Confusing causation with correlation

• Confusing numbers with rates

• Conclusion mismatch

Flawed Argument Questions

• Extreme conclusion

• Mistaking necessary for sufficient

• Guilty by association

• Unrepresentative sample

- Watch out for new words in the conclusion

Recently, a questionnaire was given to the inhabitants of Capton, Maltania’s capital city. 83% of respondents said they own one or more horses. Therefore, we can conclude that most people in Maltania own one or more horses.

(watch the entire video here)

Page 119: GMAT Critical Reasoning - everything you need to know

Flawed Argument Questions

1. Identify and summarize the conclusion and premises

2. Identify any unstated assumptions

3. Determine the primary flaw

4. Look for your description among the answer choices

5. Check all answer choices

Strategy

(watch the entire video here)

Page 120: GMAT Critical Reasoning - everything you need to know

Practice Question

A. The author assumes that the only varieties of jabberwocky are longhaired and shorthaired.

B. The author does not consider what proportion of all cases of jimmylegs involves longhaired jabberwockies.

C. The author calls into question the authenticity of the WJS data without providing any data to the contrary.

D. The author takes no account of the relative frequency of longhaired jabberwockies within the jabberwocky population.

E. The author limits the argument to an unnecessarily restrictive subset of criteria before drawing a general conclusion.

Jimmylegs, a disorder of the metatarsals, affects only jabberwockies. A popular belief contends that the longhaired variety of the jabberwocky is more predisposed to contract jimmylegs than are other varieties of jabberwocky. This, however, is clearly a myth, since data collected by the World Jabberwocky Society (WJS) indicate that, of all the jabberwockies that contract jimmylegs each year, the majority of the cases involve shorthaired jabberwockies.

Which of the following highlights the most serious flaw in the above argument?

Page 121: GMAT Critical Reasoning - everything you need to know

Practice Question

A. The author assumes that the only varieties of jabberwocky are longhaired and shorthaired.

B. The author does not consider what proportion of all cases of jimmylegs involves longhaired jabberwockies.

C. The author calls into question the authenticity of the WJS data without providing any data to the contrary.

D. The author takes no account of the relative frequency of longhaired jabberwockies within the jabberwocky population.

E. The author limits the argument to an unnecessarily restrictive subset of criteria before drawing a general conclusion.

Jimmylegs, a disorder of the metatarsals, affects only jabberwockies. A popular belief contends that the longhaired variety of the jabberwocky is more predisposed to contract jimmylegs than are other varieties of jabberwocky. This, however, is clearly a myth, since data collected by the World Jabberwocky Society (WJS) indicate that, of all the jabberwockies that contract jimmylegs each year, the majority of the cases involve shorthaired jabberwockies.

Which of the following highlights the most serious flaw in the above argument?

A: Pop. breakdown supports conclusion

Flaw: We don’t know the population breakdown

P: J-legs = jabberwocky disorder

P: Belief: longhair predisposed to j-legs

P: Majority of cases are shorthair

C: Longhair not more predisposed

(watch the entire video here)

Page 123: GMAT Critical Reasoning - everything you need to know

Goal: Find premise that resolves the paradox

Paradox Questions

Conclusion

Premise Premise Premise +

(A) New Premise (B) New Premise (C) New Premise (D) New Premise (E) New Premise

(watch the entire video here)

Page 124: GMAT Critical Reasoning - everything you need to know

Question stem examples

Identify something that resolves the contradictory information

Paradox Questions

• The paradox described above is best resolved by which of the following?

• Which of the following, if true, most helps to resolve the apparent discrepancy described above?

• Which of the following, if true, best explains the paradoxical outcome of Dr. Doolittle's experiment?

• Which one of the following most helps to explain the apparent contradiction above?

• Which one of the following, if true, most helps to explain the difference in melting points?

(watch the entire video here)

Page 125: GMAT Critical Reasoning - everything you need to know

Paradox Questions

Statistics show that the number of smokers in Maltania has steadily decreased over the past 10 years. However, during the same 10 years, the total amount of tobacco sold by Maltanian tobacco farmers has increased.

Which of the following, if true, most helps to resolve the apparent discrepancy described above?

• Looking for an “aha” premise

• Not testing ability to deconstruct arguments

• Several explanations:

- Farmers exporting to other markets

- Big increase in tobacco chewers

- and more . . .

(watch the entire video here)

Page 126: GMAT Critical Reasoning - everything you need to know

Does this explain why...?

Paradox Question strategy

1. Identify the contradictory premises

2. Explain the paradox to yourself

3. Check the answer choices while reminding yourself of the paradox

4. Check all answer choices

Paradox Questions

e.g., More tobacco sold despite fewer smokers

(watch the entire video here)

Page 127: GMAT Critical Reasoning - everything you need to know

Tips

• Keywords: yet, however, surprisingly, nonetheless, paradoxically

• Unable to identify paradox

Paradox Questions

solution unlikely

reread passage or guess and move on

• Beware of answer choices that have opposite effect

Statistics show that the number of smokers in Maltania has steadily decreased over the past 10 years. However, during the same 10 years, the total amount of tobacco sold by Maltanian tobacco farmers has increased.

Which of the following, if true, most helps to resolve the apparent discrepancy described above?

(A) The Maltanian government has introduced stop-smoking programs across the country.

(watch the entire video here)

Page 128: GMAT Critical Reasoning - everything you need to know

Practice Question

Among the mechanics at Joe’s Garage, the senior mechanics are the most adept at diagnosing engine problems. However, the itemized bills for engine repairs made during the past four years suggest otherwise. From the bills, we see that, on average, the senior mechanics took 29 minutes to correctly diagnose an engine problem, while the junior mechanics took only 22 minutes.

Which of the following, if true, most helps to resolve the apparent paradox?

A. At Joe’s Garage, mechanics with more than three years of experience are called senior mechanics.

B. Some of the junior mechanics had experience diagnosing engine problems before they began working at Joe’s Garage.

C. At Joe’s Garage, the manager typically assigns to the junior mechanics engine problems that he expects will be relatively easy to diagnose.

D. During the past 4 years, most of the senior mechanics left Joe’s Garage to open their own garages.

E. Mechanics who are more adept at diagnosing engine problems are faster

at completing the required repair.

Page 129: GMAT Critical Reasoning - everything you need to know

Practice Question

Among the mechanics at Joe’s Garage, the senior mechanics are the most adept at diagnosing engine problems. However, the itemized bills for engine repairs made during the past four years suggest otherwise. From the bills, we see that, on average, the senior mechanics took 29 minutes to correctly diagnose an engine problem, while the junior mechanics took only 22 minutes.

Which of the following, if true, most helps to resolve the apparent paradox?

P: Sr. mechs best at diagnosing

P: 29 min for sr. mechs, and .22 min for jr. mechs

Paradox: Sr. mechanics take longer despite their superior diagnostic skills.

A. At Joe’s Garage, mechanics with more than three years of experience are called senior mechanics.

B. Some of the junior mechanics had experience diagnosing engine problems before they began working at Joe’s Garage.

C. At Joe’s Garage, the manager typically assigns to the junior mechanics engine problems that he expects will be relatively easy to diagnose.

D. During the past 4 years, most of the senior mechanics left Joe’s Garage to open their own garages.

E. Mechanics who are more adept at diagnosing engine problems are faster

at completing the required repair.

(watch the entire video here)

Page 130: GMAT Critical Reasoning - everything you need to know

Evaluate the Conclusion Questions (watch the entire video here)

Page 131: GMAT Critical Reasoning - everything you need to know

(A) Question (B) Question (C) Question (D) Question (E) Question

Conclusion

Premise Premise Premise

Assumption Assumption +

Evaluate the Conclusion Questions

Goal: Find the question that, when answered, best helps to evaluate the conclusion.

(watch the entire video here)

Page 132: GMAT Critical Reasoning - everything you need to know

Researcher: Two years ago, a wolf pack was relocated to Bilford Island. Although the local rabbit population has decreased drastically since the relocation, the wolves are not to blame for this decrease. Our study shows that the unprecedented number of recent rabbit deaths is due to the myxoma virus.

Which of the following would be most relevant to investigate in order to evaluate the researcher’s conclusion?

Evaluate the Conclusion Questions

Conclusion: Wolves not responsible for population decrease.

A) In what season was the wolf pack relocated to Bilford Island?

B) What proportion of rabbits have died from the myxoma virus?

C) Did the wolf pack introduce the myxoma virus to Bilford Island?

D) . . . Goal: Find the question that, when answered,

helps evaluate the conclusion

(watch the entire video here)

Page 133: GMAT Critical Reasoning - everything you need to know

Question stem examples

Identify a question that would help gauge the strength of the conclusion

Evaluate the Conclusion Questions

• Knowing which of the following would be most useful in evaluating the argument?

• Which of the following would be most relevant to investigate in order to evaluate the researcher’s conclusion?

• Clarification of which of the following issues would be most important to evaluating the spokesperson’s position?

(watch the entire video here)

Page 134: GMAT Critical Reasoning - everything you need to know

Strategy

1. Identify and summarize the conclusion and premises

2. Identify any assumptions

3. Check the answer choices by providing an answer to each question and relating it to the conclusion

4. Check all answer choices

Evaluate the Conclusion Questions (watch the entire video here)

Page 135: GMAT Critical Reasoning - everything you need to know

Evaluate the Conclusion Questions

Goal: Find the question that, when answered, helps evaluate the conclusion.

3. Check the answer choices by providing an answer to each question and relating it to the conclusion

(watch the entire video here)

Page 136: GMAT Critical Reasoning - everything you need to know

Evaluate the Conclusion Questions

Conclusion: Wolves not responsible for population decrease.

3. Check the answer choices by providing an answer to each question and relating it to the conclusion

A) In what season was the wolf pack relocated to Bilford Island?

Researcher: Two years ago, a wolf pack was relocated to Bilford Island. Although the local rabbit population has decreased drastically since the relocation, the wolves are not to blame for this decrease. Our study shows that the unprecedented number of recent rabbit deaths is due to the myxoma virus.

Which of the following would be most relevant to investigate to evaluate the researcher’s conclusion?

Summer

(watch the entire video here)

Page 137: GMAT Critical Reasoning - everything you need to know

Evaluate the Conclusion Questions

Conclusion: Wolves not responsible for population decrease.

3. Check the answer choices by providing an answer to each question and relating it to the conclusion

A) In what season was the wolf pack relocated to Bilford Island?

Researcher: Two years ago, a wolf pack was relocated to Bilford Island. Although the local rabbit population has decreased drastically since the relocation, the wolves are not to blame for this decrease. Our study shows that the unprecedented number of recent rabbit deaths is due to the myxoma virus.

Which of the following would be most relevant to investigate to evaluate the researcher’s conclusion?

Winter

(watch the entire video here)

Page 138: GMAT Critical Reasoning - everything you need to know

Evaluate the Conclusion Questions

Conclusion: Wolves not responsible for population decrease.

3. Check the answer choices by providing an answer to each question and relating it to the conclusion

A) In what season was the wolf pack relocated to Bilford Island?

B) What proportion of rabbits have died from the myxoma virus?

Researcher: Two years ago, a wolf pack was relocated to Bilford Island. Although the local rabbit population has decreased drastically since the relocation, the wolves are not to blame for this decrease. Our study shows that the unprecedented number of recent rabbit deaths is due to the myxoma virus.

Which of the following would be most relevant to investigate to evaluate the researcher’s conclusion?

43%

Winter

(watch the entire video here)

Page 139: GMAT Critical Reasoning - everything you need to know

Evaluate the Conclusion Questions

Conclusion: Wolves not responsible for population decrease.

3. Check the answer choices by providing an answer to each question and relating it to the conclusion

A) In what season was the wolf pack relocated to Bilford Island?

B) What proportion of rabbits have died from the myxoma virus?

C) Did the wolf pack introduce the myxoma virus to Bilford Island?

Researcher: Two years ago, a wolf pack was relocated to Bilford Island. Although the local rabbit population has decreased drastically since the relocation, the wolves are not to blame for this decrease. Our study shows that the unprecedented number of recent rabbit deaths is due to the myxoma virus.

Which of the following would be most relevant to investigate to evaluate the researcher’s conclusion?

43%

Winter

Yes

(watch the entire video here)

Page 140: GMAT Critical Reasoning - everything you need to know

Evaluate the Conclusion Questions

Conclusion: Wolves not responsible for population decrease.

3. Check the answer choices by providing an answer to each question and relating it to the conclusion

A) In what season was the wolf pack relocated to Bilford Island?

B) What proportion of rabbits have died from the myxoma virus?

C) Did the wolf pack introduce the myxoma virus to Bilford Island?

Researcher: Two years ago, a wolf pack was relocated to Bilford Island. Although the local rabbit population has decreased drastically since the relocation, the wolves are not to blame for this decrease. Our study shows that the unprecedented number of recent rabbit deaths is due to the myxoma virus.

Which of the following would be most relevant to investigate to evaluate the researcher’s conclusion?

43%

Winter

No

(watch the entire video here)

Page 141: GMAT Critical Reasoning - everything you need to know

Evaluate the Conclusion Questions

Strategy

1. Identify and summarize the conclusion and premises

2. Identify any assumptions

3. Check the answer choices by providing an answer to each question and relating it to the conclusion

4. Check all answer choices

(watch the entire video here)

Page 142: GMAT Critical Reasoning - everything you need to know

Practice Question

Professor Gelding: The belief that our planet will experience a wave of extreme weather events over the next 100 years is nonsense. There is no evidence to support that the rate of extreme weather events is increasing.

Professor Brink: To see where you are mistaken, you need only examine the data concerning Category 5 hurricanes in the Gulf of Mexico that made landfall in the past 100 years. In the first 50 years, there were only 6 such hurricanes, but in the last 50 years, there were 19 such hurricanes.

Clarification of which of the following issues would be most important to an evaluation of Professor Brink’s conclusion?

A. Does the rate at which Category 5 hurricanes make landfall affect the rate of other extreme weather events?

B. Do the statistics concerning Category 5 hurricanes that made landfall represent extreme weather events in the Gulf of Mexico?

C. During the last 100 years, what percent of hurricanes made landfall?

D. How many extreme weather events occurred in the past 100 years?

E. Do the Category 5 hurricanes in the Gulf of Mexico that made landfall in the past 100 years represent extreme weather events in general?

Page 143: GMAT Critical Reasoning - everything you need to know

Practice Question

A. Does the rate at which Category 5 hurricanes make landfall affect the rate of other extreme weather events?

B. Do the statistics concerning Category 5 hurricanes that made landfall represent extreme weather events in the Gulf of Mexico?

C. During the last 100 years, what percent of hurricanes made landfall?

D. How many extreme weather events occurred in the past 100 years?

E. Do the Category 5 hurricanes in the Gulf of Mexico that made landfall in the past 100 years represent extreme weather events in general?

A: hurricane rate characterizes extreme weather rate

P: 1st 50 yrs: 6 hurricanes

P: 2nd 50 yrs: 19 hurricanes

C: EW rate is increasing

Professor Gelding: The belief that our planet will experience a wave of extreme weather events over the next 100 years is nonsense. There is no evidence to support that the rate of extreme weather events is increasing.

Professor Brink: To see where you are mistaken, you need only examine the data concerning Category 5 hurricanes in the Gulf of Mexico that made landfall in the past 100 years. In the first 50 years, there were only 6 such hurricanes, but in the last 50 years, there were 19 such hurricanes.

Clarification of which of the following issues would be most important to an evaluation of Professor Brink’s conclusion?

(watch the entire video here)

Page 145: GMAT Critical Reasoning - everything you need to know

Miscellaneous Tips

• Question type frequencies

• GMAT words

• EXCEPT questions

• Being aggressive

(watch the entire video here)

Page 146: GMAT Critical Reasoning - everything you need to know

Question type frequencies

Miscellaneous Tips

1. Weaken the Argument

2. Strengthen the Argument

3. Assumption

4. Conclusion/Inference

5. Method of Reasoning

6. Flawed Argument

7. Evaluation

8. Paradox

(watch the entire video here)

Page 147: GMAT Critical Reasoning - everything you need to know

Question type frequencies

Miscellaneous Tips

1. Weaken the Argument

2. Strengthen the Argument

3. Assumption

4. Conclusion/Inference

5. Method of Reasoning

6. Flawed Argument

7. Evaluation

8. Paradox

¾+

(watch the entire video here)

Page 148: GMAT Critical Reasoning - everything you need to know

Miscellaneous Tips

GMAT words

Everybody likes ice cream = a lot of people like ice cream

= most people like ice cream

Common usage

Everybody likes ice cream = every person likes ice cream

GMAT usage

(watch the entire video here)

Page 149: GMAT Critical Reasoning - everything you need to know

Miscellaneous Tips

GMAT words

• Read words in their strongest, most literal sense

all, none, everyone, no one, always, never, each every, anywhere, nowhere

• Some: 1 or more

Some Gigacorp employees are college graduates.

Some countries in Europe are named Italy.

Some of Earth’s oceans contain salt water

• Most: More than 50%

Most of Earth’s oceans contain salt water

(watch the entire video here)

Page 150: GMAT Critical Reasoning - everything you need to know

Miscellaneous Tips

EXCEPT questions

Today, Gary did not arrive at the office until 11am. Therefore Gary will be fired.

Each of the following, if true, weakens the conclusion above EXCEPT

• Does not mean “strengthen” the conclusion

(watch the entire video here)

Page 151: GMAT Critical Reasoning - everything you need to know

Miscellaneous Tips

EXCEPT questions

Each of the following people is a former President of the United States EXCEPT

If the person is a former US president eliminate it.

(A) Abraham Lincoln

(B) George Washington

(C) Bugs Bunny

(D) Jimmy Carter

(E) George Bush

(watch the entire video here)

Page 152: GMAT Critical Reasoning - everything you need to know

Miscellaneous Tips

EXCEPT questions

Today, Gary did not arrive at the office until 11am. Therefore Gary will be fired.

Each of the following, if true, weakens the conclusion above EXCEPT

If it weakens the conclusion, eliminate it.

(A) Work begins at noon

(B) Gary is self employed

(C) Gary likes ice cream

(watch the entire video here)

Page 153: GMAT Critical Reasoning - everything you need to know

Miscellaneous Tips

Being aggressive

• Critical reasoning questions can be time consuming

• Students get lost in the answer choices

• Be aggressive!

“What if I eliminate all answer choices?”

• This will not happen often

• You will have earned time to recheck answer choices

Look for reasons to reject answer choices

(watch the entire video here)

Page 154: GMAT Critical Reasoning - everything you need to know

Miscellaneous Tips

• Question type frequencies

• GMAT words

• EXCEPT questions

• Being aggressive

(watch the entire video here)

Page 155: GMAT Critical Reasoning - everything you need to know

Practice Question

For decades, countries have measured wheat field productivity using the Wheat Field Productivity Index (WFPI), which is equal to the average number of pounds of grain harvested per acre of wheat. In 2005, Maltania’s WFPI was 60 pounds per acre more than Italy’s WFPI. In 2006, Maltania’s WFPI was 75 pounds per acre more than Italy’s. Therefore, Maltania’s wheat field productivity must have increased during that period.

Which of the following, if true, provides the strongest support for the conclusion above?

A. Between 2005 and 2006, the number of acres of wheat planted in both Maltania and Italy increased at the same rate.

B. In 2006, Maltania received 20% more sunshine than usual.

C. Between 2005 and 2006, Italy’s wheat field productivity decreased by 10 pounds per acre.

D. In 2006, Italy experienced its worst drought in decades.

E. In 2006, Maltania began subsidizing all of its wheat farmers.

Page 156: GMAT Critical Reasoning - everything you need to know

Practice Question

P: WFPI = ave lbs wheat/acre

P: 2005: Maltania’s WFPI 60 more than Italy’s

P: 2006: Maltania’s WFPI 75 more than Italy’s

C: Maltania’s WFPI

A. Between 2005 and 2006, the number of acres of wheat planted in both Maltania and Italy increased at the same rate.

B. In 2006, Maltania received 20% more sunshine than usual.

C. Between 2005 and 2006, Italy’s wheat field productivity decreased by 10 pounds per acre.

D. In 2006, Italy experienced its worst drought in decades.

E. In 2006, Maltania began subsidizing all of its wheat farmers.

For decades, countries have measured wheat field productivity using the Wheat Field Productivity Index (WFPI), which is equal to the average number of pounds of grain harvested per acre of wheat. In 2005, Maltania’s WFPI was 60 pounds per acre more than Italy’s WFPI. In 2006, Maltania’s WFPI was 75 pounds per acre more than Italy’s. Therefore, Maltania’s wheat field productivity must have increased during that period.

Which of the following, if true, provides the strongest support for the conclusion above?

(watch the entire video here)

Page 157: GMAT Critical Reasoning - everything you need to know

Practice Question

All of Pedro’s friends say they know someone who is a licensed circus clown. Pedro does not know any licensed circus clowns, and he is not unique among his friends in this respect.

If the above statements are true, which of the following must also be true?

A. Some of Pedro’s friends are lying.

B. Most licensed circus clowns lie about their profession.

C. Some of Pedro’s friends have friends that Pedro is not friends with.

D. Pedro is a licensed circus clown.

E. All of Pedro’s friends know the same licensed circus clown.

Page 158: GMAT Critical Reasoning - everything you need to know

Practice Question

All of Pedro’s friends say they know someone who is a licensed circus clown. Pedro does not know any licensed circus clowns, and he is not unique among his friends in this respect.

If the above statements are true, which of the following must also be true?

A. Some of Pedro’s friends are lying.

B. Most licensed circus clowns lie about their profession.

C. Some of Pedro’s friends have friends that Pedro is not friends with.

D. Pedro is a licensed circus clown.

E. All of Pedro’s friends know the same licensed circus clown.

P: All friends SAY they know a C

P: Pedro knows zero Cs

P: Some friends don’t know a C

(watch the entire video here)

Page 159: GMAT Critical Reasoning - everything you need to know

Practice Question

Reporter: The number of foreigners working at US ski resorts was lower last year than it had been in previous years. Last year, several ski resorts hired more local residents than they had hired in previous years, so it is likely that the reduced number of foreign workers at US ski resorts last year was due to the decreased demand for their services.

Which of the following, if true, most seriously undermines the reporter’s argument?

A. All foreigners who received work visas for last year’s ski season were able to find work at US ski resorts.

B. The average wait time to receive a US work visa has decreased substantially over the past three years.

C. The number of ski resorts that hired foreigners last year was the same as in previous years.

D. Foreigners can work for more than one ski resort in any given year.

E. Recent changes to US immigration laws drastically decreased the total number of work visas issued for last year.

Page 160: GMAT Critical Reasoning - everything you need to know

Practice Question

Reporter: The number of foreigners working at US ski resorts was lower last year than it had been in previous years. Last year, several ski resorts hired more local residents than they had hired in previous years, so it is likely that the reduced number of foreign workers at US ski resorts last year was due to the decreased demand for their services.

Which of the following, if true, most seriously undermines the reporter’s argument?

A. All foreigners who received work visas for last year’s ski season were able to find work at US ski resorts.

B. The average wait time to receive a US work visa has decreased substantially over the past three years.

C. The number of ski resorts that hired foreigners last year was the same as in previous years.

D. Foreigners can work for more than one ski resort in any given year.

E. Recent changes to US immigration laws drastically decreased the total number of work visas issued for last year.

A: Nothing else caused decrease

A: Fewer foreign workers did not cause increase in locals hired

P: # foreign wrkrs lower last yr

P: More locals hired last yr

C: local employees caused foreign employees

(watch the entire video here)

Page 161: GMAT Critical Reasoning - everything you need to know

Practice Question

After a plague of locusts attacked every crop in Baker County, Kevin Kevinson claimed that the severity of damage to his crops was due to the fish-based fertilizer he applied to his fields before planting.

Which of the following investigations is most likely to yield significant information that would help evaluate Kevin’s argument?

A. Comparing the value of Kevin’s crop damage to the average value of the crop damage at farms where fish-based fertilizer was not applied to the fields before planting

B. Determining the extent to which fish-based fertilizer increases crop yield

C. Comparing the long-term effects of Kevin’s crop damage to the long-term effects of other crop damage in Baker County

D. Comparing the severity of crop damage at Kevin’s farm with the crop damage at agriculturally similar Baker County farms where fish-based fertilizer was not applied to fields before planting

E. Determining the percentage of Baker County farms that applied fish-based fertilizer to their fields before planting

Page 162: GMAT Critical Reasoning - everything you need to know

A. Comparing the value of Kevin’s crop damage to the average value of the crop damage at farms where fish-based fertilizer was not applied to the fields before planting

B. Determining the extent to which fish-based fertilizer increases crop yield

C. Comparing the long-term effects of Kevin’s crop damage to the long-term effects of other crop damage in Baker County

D. Comparing the severity of crop damage at Kevin’s farm with the crop damage at agriculturally similar Baker County farms where fish-based fertilizer was not applied to fields before planting

E. Determining the percentage of Baker County farms that applied fish-based fertilizer to their fields before planting

Practice Question

After a plague of locusts attacked every crop in Baker County, Kevin Kevinson claimed that the severity of damage to his crops was due to the fish-based fertilizer he applied to his fields before planting.

Which of the following investigations is most likely to yield significant information that would help evaluate Kevin’s argument?

P: Locusts hit all crops

P: Kevin applied FBF to crops

C: FBF caused severity

A: Nothing else caused the severity

(watch the entire video here)

Page 163: GMAT Critical Reasoning - everything you need to know

Practice Question

Ayla is taller than Elan, and Brek is the same height as Diego. Since Chayna is shorter than Diego, it follows that Ayla is taller than Chayna.

Each of the following, when added to the argument as an additional premise, makes the argument logically correct EXCEPT:

A. Diego is shorter than Elan.

B. Ayla and Diego are the same height.

C. Brek is taller than Elan.

D. Elan and Chayna are the same height.

E. Brek is shorter than Ayla.

Page 164: GMAT Critical Reasoning - everything you need to know

Practice Question

A. Diego is shorter than Elan.

B. Ayla and Diego are the same height.

C. Brek is taller than Elan.

D. Elan and Chayna are the same height.

E. Brek is shorter than Ayla.

If it makes the argument correct, eliminate it.

A

C

Premises:

Conclusion:

+ B=D

C

A

E

Ayla is taller than Elan, and Brek is the same height as Diego. Since Chayna is shorter than Diego, it follows that Ayla is taller than Chayna.

Each of the following, when added to the argument as an additional premise, makes the argument logically correct EXCEPT:

(watch the entire video here)

Page 165: GMAT Critical Reasoning - everything you need to know

Practice Question

A. In the past 7 years, the CPT for king crabs has decreased by 50%.

B. 8 years ago, commercial crab-fishing boats began using high-tech sonar equipment that enables them to locate crabs with much greater accuracy.

C. 9 years ago, a deadly species of sea lice drove many crab species to near extinction.

D. In the past 10 years, the number of commercial crab traps set each year has remained relatively constant.

E. In the past 10 years, the worldwide price of snow crabs has more than doubled.

To estimate changes in crab populations, biologists monitor what is known as “crabs per trap” (CPT). The CPT is the average number of crabs that commercial crab fishermen catch per crab trap. Since the current CPT for snow crabs is the same as it was 10 years ago, biologists conclude that today’s snow crab population is approximately the same as it was 10 years ago.

Which of the following, if true, most seriously weakens the argument?

Page 166: GMAT Critical Reasoning - everything you need to know

Practice Question

A. In the past 7 years, the CPT for king crabs has decreased by 50%.

B. 8 years ago, commercial crab-fishing boats began using high-tech sonar equipment that enables them to locate crabs with much greater accuracy.

C. 9 years ago, a deadly species of sea lice drove many crab species to near extinction.

D. In the past 10 years, the number of commercial crab traps set each year has remained relatively constant.

E. In the past 10 years, the worldwide price of snow crabs has more than doubled.

To estimate changes in crab populations, biologists monitor what is known as “crabs per trap” (CPT). The CPT is the average number of crabs that commercial crab fishermen catch per crab trap. Since the current CPT for snow crabs is the same as it was 10 years ago, biologists conclude that today’s snow crab population is approximately the same as it was 10 years ago.

Which of the following, if true, most seriously weakens the argument?

P: CPT = ave # crabs/trap

P: CPT now = 10 yrs ago

C: SC pop. ≈ 10 yrs ago

A: CPT reflects pop.

(watch the entire video here)

Page 167: GMAT Critical Reasoning - everything you need to know

Practice Question

Identical twins have livers that are genetically identical. Scientists recently discovered that when one twin has hepachrinosis, a debilitating liver disease, the caudate lobe of the affected twin’s liver is significantly smaller than the caudate lobe of the unaffected twin’s liver. No such difference is found when neither twin has hepachrinosis. From this information, it can be concluded that hepachrinosis is caused by diminished liver capacity.

Which of the following is an assumption required by the argument?

A. Many diseases are caused by diminished organ capacity.

B. The caudate lobe of a healthy identical twin is the same size as the caudate lobe of a healthy person who is not an identical twin.

C. When two identical twins both suffer from hepachrinosis, their caudate lobes are the same size.

D. The relative smallness of the caudate lobe of an affected person is not the result of medications used in the treatment of hepachrinosis.

E. People who have an identical twin are no more likely to contract hepachrinosis than people who do not have an identical twin.

Page 168: GMAT Critical Reasoning - everything you need to know

Practice Question

Identical twins have livers that are genetically identical. Scientists recently discovered that when one twin has hepachrinosis, a debilitating liver disease, the caudate lobe of the affected twin’s liver is significantly smaller than the caudate lobe of the unaffected twin’s liver. No such difference is found when neither twin has hepachrinosis. From this information, it can be concluded that hepachrinosis is caused by diminished liver capacity.

Which of the following is an assumption required by the argument?

A. Many diseases are caused by diminished organ capacity.

B. The caudate lobe of a healthy identical twin is the same size as the caudate lobe of a healthy person who is not an identical twin.

C. When two identical twins both suffer from hepachrinosis, their caudate lobes are the same size.

D. The relative smallness of the caudate lobe of an affected person is not the result of medications used in the treatment of hepachrinosis.

E. People who have an identical twin are no more likely to contract hepachrinosis than people who do not have an identical twin.

P: Twins have = livers

P: Lobe of affected twin is smaller

P: Lobes of unaffected twins are =

C: Small lobe causes H

A: nothing else causes H

(watch the entire video here)

Page 169: GMAT Critical Reasoning - everything you need to know

Practice Question

On average, when 100 non-pregnant women take the Pregunda home pregnancy test, 10 of them will test positive. That is, the test will indicate that the women are pregnant. Conversely, when 100 pregnant women take the Pregunda home pregnancy test, 99 of them will test positive. Therefore, if a group of women take the Pregunda home pregnancy test, the vast majority of those who test positive will be pregnant.

The above argument is flawed primarily because it

A. does not take into consideration the test’s margin of error

B. suggests that non-pregnant women are more likely to test positive than pregnant women are

C. presupposes the proportion of pregnant women in the group

D. assumes that the test’s accuracy is not affected by other factors

E. ignores the fact that women who do not test positive may be pregnant

Page 170: GMAT Critical Reasoning - everything you need to know

Practice Question

A. does not take into consideration the test’s margin of error

B. suggests that non-pregnant women are more likely to test positive than pregnant women are

C. presupposes the proportion of pregnant women in the group

D. assumes that the test’s accuracy is not affected by other factors

E. ignores the fact that women who do not test positive may be pregnant

On average, when 100 non-pregnant women take the Pregunda home pregnancy test, 10 of them will test positive. That is, the test will indicate that the women are pregnant. Conversely, when 100 pregnant women take the Pregunda home pregnancy test, 99 of them will test positive. Therefore, if a group of women take the Pregunda home pregnancy test, the vast majority of those who test positive will be pregnant.

The above argument is flawed primarily because it

A: Test accuracy is constant

P: 10% of NP test positive

P: 99% of P test positive

C: Group majority of positive are pregnant

(watch the entire video here)

Page 171: GMAT Critical Reasoning - everything you need to know

Practice Question

Jesse: The H1 vaccine prevents people from contracting thramboxia, a disease that can cause short-term paralysis. While thramboxia is an unpleasant disease with paralysis lasting up to 3 months, the H1 vaccine is not worth getting since hundreds of people die each year from the H1 vaccination.

Harjit: But thousands of people die each year as a direct result of contracting thramboxia. So, in addition to preventing thramboxia, the H1 vaccine prevents deaths.

Harjit responds to Jesse by

A. questioning the truth of a statement on which Jesse’s conclusion is based

B. suggesting that Jesse’s argument overlooks an important consequence

C. noting that Jesse’s argument contradicts that which he is trying to prove

D. reinforcing Jesse’s conclusion before suggesting a new conclusion

E. citing evidence that disproves the evidence cited by Jesse in drawing his conclusion

Page 172: GMAT Critical Reasoning - everything you need to know

Practice Question

Jesse: The H1 vaccine prevents people from contracting thramboxia, a disease that can cause short-term paralysis. While thramboxia is an unpleasant disease with paralysis lasting up to 3 months, the H1 vaccine is not worth getting since hundreds of people die each year from the H1 vaccination.

Harjit: But thousands of people die each year as a direct result of contracting thramboxia. So, in addition to preventing thramboxia, the H1 vaccine prevents deaths.

Harjit responds to Jesse by

A. questioning the truth of a statement on which Jesse’s conclusion is based

B. suggesting that Jesse’s argument overlooks an important consequence

C. noting that Jesse’s argument contradicts that which he is trying to prove

D. reinforcing Jesse’s conclusion before suggesting a new conclusion

E. citing evidence that disproves the evidence cited by Jesse in drawing his conclusion

P: H1 prevents T

P: T can cause paralysis

P: 100s die from H1

C: Don’t get H1 vaccine

P: 1000’s die from T

P: H1 prevents T & deaths

C: Get H1 vaccine

There’s also Y. My option accomplishes 2 things

(watch the entire video here)

Page 173: GMAT Critical Reasoning - everything you need to know

Practice Question

A. Very few people living in poorer communities are employed in the bakeries that make the bread.

B. In poorer communities, there are very few food alternatives to bread.

C. In poorer communities, bread is often the least expensive staple food, so when its price increases, people have less money to spend on other, more expensive foods.

D. Most people in poorer communities are unlikely to change their eating habits.

E. During some holidays, the consumption of food increases substantially.

For most products there exists an inverse relationship between price and consumption; if the price of a product increases, then consumption of that product decreases, and if the price decreases, consumption increases. However, in some poorer communities, when the price of bread increases, the consumption of bread increases.

Which of the following, if true, does the most to explain the paradoxical scenario?

Page 174: GMAT Critical Reasoning - everything you need to know

Practice Question

P: $ then buying

P: Bread $ then buying

Paradox: People buy more bread when they should buy less

For most products there exists an inverse relationship between price and consumption; if the price of a product increases, then consumption of that product decreases, and if the price decreases, consumption increases. However, in some poorer communities, when the price of bread increases, the consumption of bread increases.

Which of the following, if true, does the most to explain the paradoxical scenario?

A. Very few people living in poorer communities are employed in the bakeries that make the bread.

B. In poorer communities, there are very few food alternatives to bread.

C. In poorer communities, bread is often the least expensive staple food, so when its price increases, people have less money to spend on other, more expensive foods.

D. Most people in poorer communities are unlikely to change their eating habits.

E. During some holidays, the consumption of food increases substantially.

Giffen good: A consumer good for which

consumption increases when price increases

(watch the entire video here)

Page 175: GMAT Critical Reasoning - everything you need to know

Practice Question

It is very difficult for piano retailers to grow in the piano market. Despite population increases, the demand for pianos has remained constant for decades. Furthermore, when people decide to buy a piano, they typically go to the nearest piano retailer to make their purchase. Given this environment, a piano retailer can gain market share in the piano market only through purchasing its competitors, and not through aggressive marketing strategies.

Which of the following, if true, contributes most strongly to a refutation of the conclusion above?

A. Of the four mergers of piano retailers over the past six years, three led to a decline in profits for the newly-merged companies.

B. Through staff layoffs and warehouse closures, some of the larger piano retailers have achieved price reductions that are causing the smaller piano retailers to go out of business.

C. Most piano retailers sell a variety of products other than pianos.

D. Many public schools, traditionally responsible for almost half of all piano purchases, are eliminating their music programs in the face of severe budget cuts.

E. Several piano retailers have used aggressive marketing strategies such as publicity events, direct mail and in-store financing to increase customer loyalty.

Page 176: GMAT Critical Reasoning - everything you need to know

Practice Question

It is very difficult for piano retailers to grow in the piano market. Despite population increases, the demand for pianos has remained constant for decades. Furthermore, when people decide to buy a piano, they typically go to the nearest piano retailer to make their purchase. Given this environment, a piano retailer can gain market share in the piano market only through purchasing its competitors, and not through aggressive marketing strategies.

Which of the following, if true, contributes most strongly to a refutation of the conclusion above?

A. Of the four mergers of piano retailers over the past six years, three led to a decline in profits for the newly-merged companies.

B. Through staff layoffs and warehouse closures, some of the larger piano retailers have achieved price reductions that are causing the smaller piano retailers to go out of business.

C. Most piano retailers sell a variety of products other than pianos.

D. Many public schools, traditionally responsible for almost half of all piano purchases, are eliminating their music programs in the face of severe budget cuts.

E. Several piano retailers have used aggressive marketing strategies such as publicity events, direct mail and in-store financing to increase customer loyalty.

P: Hard for retailer to grow

P: Demand constant

P: People buy from closest

C: share only through buying competitors

A: No other way to share

(watch the entire video here)

Page 177: GMAT Critical Reasoning - everything you need to know

Practice Question

Maltania ranks far behind Finland and Sweden when it comes to employee job satisfaction. In all three countries, revenue-sharing programs and onsite daycare successfully increase job satisfaction. In Finland and Sweden, labor laws require companies with more than 10 employees to have revenue-sharing programs and onsite daycare. In Maltania, however, revenue-sharing programs and onsite daycare are found in only a few companies that have voluntarily established them.

Which of the following can be inferred from the statements above?

A. In Finland and Sweden, prior to the introduction of labor laws, the level of employee job satisfaction was similar to that in Maltania.

B. Revenue-sharing programs and onsite daycare are no more effective in increasing employee job satisfaction when they are established voluntarily than when they are established to comply with labor laws.

C. Establishing revenue-sharing programs and onsite daycare at all Maltanian companies with over 10 employees would increase employee job satisfaction in Maltania.

D. Companies with high job satisfaction typically have revenue-sharing programs.

E. Prior to the introduction of labor laws in Finland and Sweden, most companies with more than 10 employees did not have revenue-sharing programs and onsite daycare.

Page 178: GMAT Critical Reasoning - everything you need to know

Practice Question

Maltania ranks far behind Finland and Sweden when it comes to employee job satisfaction. In all three countries, revenue-sharing programs and onsite daycare successfully increase job satisfaction. In Finland and Sweden, labor laws require companies with more than 10 employees to have revenue-sharing programs and onsite daycare. In Maltania, however, revenue-sharing programs and onsite daycare are found in only a few companies that have voluntarily established them.

Which of the following can be inferred from the statements above?

P: M behind F&S in JS

P: RS & DC JS

P: F&S must have RS & DC

P: M has voluntary RS & DC

A. In Finland and Sweden, prior to the introduction of labor laws, the level of employee job satisfaction was similar to that in Maltania.

B. Revenue-sharing programs and onsite daycare are no more effective in increasing employee job satisfaction when they are established voluntarily than when they are established to comply with labor laws.

C. Establishing revenue-sharing programs and onsite daycare at all Maltanian companies with over 10 employees would increase employee job satisfaction in Maltania.

D. Companies with high job satisfaction typically have revenue-sharing programs.

E. Prior to the introduction of labor laws in Finland and Sweden, most companies with more than 10 employees did not have revenue-sharing programs and onsite daycare.

(watch the entire video here)

Page 179: GMAT Critical Reasoning - everything you need to know

Practice Question

Representative for the Maltanian Association of Cyclists: Over 1000 cyclists died in accidents last year. Accident reports indicate that 80% of the victims were not wearing a helmet at the time of their accident. This information indicates that, by wearing a helmet, cyclists can reduce their risk of dying if they are involved in an accident.

Which of the following, if true, best supports the representative’s argument?

A) The Maltanian Association of Cyclists is not funded by any helmet manufacturers.

B) Maltania has laws requiring all cyclists to wear a helmet.

C) Last year, less than 20% of the cyclists involved in accidents were not wearing a helmet at the time of the accident.

D) Most of the accidents in Maltania involve a cyclist and a motor vehicle.

E) Most of the cyclists who survived an accident last year indicate that their injuries would have been much worse had they not been wearing a helmet at the time.

Page 180: GMAT Critical Reasoning - everything you need to know

Practice Question

Representative for the Maltanian Association of Cyclists: Over 1000 cyclists died in accidents last year. Accident reports indicate that 80% of the victims were not wearing a helmet at the time of their accident. This information indicates that, by wearing a helmet, cyclists can reduce their risk of dying if they are involved in an accident.

Which of the following, if true, best supports the representative’s argument?

P: 1000+ cyclist deaths

P: 80% of victims no helmet

C: Helmets save lives

A: Less than 80% of cyclists in accidents were not wearing a helmet

A) The Maltanian Association of Cyclists is not funded by any helmet manufacturers.

B) Maltania has laws requiring all cyclists to wear a helmet.

C) Last year, less than 20% of the cyclists involved in accidents were not wearing a helmet at the time of the accident.

D) Most of the accidents in Maltania involve a cyclist and a motor vehicle.

E) Most of the cyclists who survived an accident last year indicate that their injuries would have been much worse had they not been wearing a helmet at the time.

(watch the entire video here)

Page 181: GMAT Critical Reasoning - everything you need to know

For additional practice questions, see the bottom of our Critical Reasoning module

www.GMATPrepNow.com

GMAT Critical Reasoning - Everything you need to know

Page 182: GMAT Critical Reasoning - everything you need to know

If you enjoyed this learning format, let us know, and we’ll add similar resources to our

SlideShare page.

GMAT Critical Reasoning - Everything you need to know